Fiche de mathématiques
> >

Baccalauréat S
Session Juin 2015

Partager :
Durée de l'épreuve : 4 heures
Coefficient 7 (non spé) / Coefficient 9 (spé)


Calculatrices électroniques autorisées


6 points

exercice 1 Commun à tous les candidats


Cet exercice traite d'un variable aléatoire suivant une loi exponentielle.
Il s'agira de vérifier des affirmations concernant des probabilités puis enfin de donner l'espérance de la variable aléatoire étudiée.
Ensuite, il faudra calculer la probabilité de certains événements.
Sujet de Mathématiques Bac S 2015 - Spé et non Spé : image 1

Dans la deuxième partie de l'exercice, on étudiera un cas pratique appliqué à une chaîne de magasins souhaitant fidéliser ses clients par le biais de bons d'achats.
Il faudra alors calculer les probabilités d'obtenir certains type de bons d'achat en fonction du profil de client.

Sujet de Mathématiques Bac S 2015 - Spé et non Spé : image 2


Sujet de Mathématiques Bac S 2015 - Spé et non Spé : image 3


3 points

exercice 2 Commun à tous les candidats


Nous sommes en présence d'un repère orthonormé et de 4 points dont on connaît les coordonnées. On défini ensuite des points qui se déplacent sur les droites passant par les points fixes définis plus tôt, avec des vitesses constantes. Il s'agira de répondre à plusieurs questions, vérifiant notamment si certaines droites sont parallèles ou sécantes.

Sujet de Mathématiques Bac S 2015 - Spé et non Spé : image 4


5 points

exercice 3 Uniquement pour les élèves n'ayant pas suivi l'enseignement de spécialité


Il faut dans cet exercice résoudre dans l'ensemble C des nombres complexes l'équaction (E) d'inconnue z telle que :
Z²-8z+64=0

Sujet de Mathématiques Bac S 2015 - Spé et non Spé : image 5


5 points

exercice 3 Uniquement pour les élèves ayant suivi l'enseignement de spécialité



Sujet de Mathématiques Bac S 2015 - Spé et non Spé : image 6


Sujet de Mathématiques Bac S 2015 - Spé et non Spé : image 7


6 points

exercice 4 Commun à tous les candidats



Sujet de Mathématiques Bac S 2015 - Spé et non Spé : image 8


Sujet de Mathématiques Bac S 2015 - Spé et non Spé : image 9









exercice 1 - COMMUN À TOUS LES CANDIDATS


Tous les résultats sont donnés à 10^{-3} près.

Partie 1


1. a. X suit une la loi exponentielle de paramètre \lambda donc, pour 0\le c<d,
P(c\le X\le d)=\displaystyle\int_c^df(x)\text{d}x=\int_c^d\lambda e^{-\lambda x}\text{d} x=\left[-e^{-\lambda x}\right]_c^d=e^{-\lambda c}-e^{-\lambda d}.


1. b. L'événement X>20 est l'événement contraire (ou complémentaire) de l'événement X\le20 donc en appliquant le résultat démontré en 1. a.
\begin{array}{rcl} P(X>20)&=&1-P(0\le X\le20)\\\\ &=&1-(e^0-e^{-20\lambda})\\\\ &=&e^{-20\lambda} \end{array}

On cherche donc à déterminer \lambda tel que e^{-20\lambda}=0,05 soit :

-20\lambda=\ln(0,05)

soit
\lambda=-\dfrac{\ln(0,05)}{20}\approx0,150


1. c. D'après le cours, l'espérance de cette loi exponentielle est \mathbb{E}(X)=\cfrac{1}{\lambda} \approx6,667. (si le calcul est fait avec la valeur approximative trouvée précédemment, ou 6,676 si le calcul est réalisé avec la valeur exacte de \lambda).

1. d. D'après la question 1.a,
P(10\le X\le20)=e^{-10\lambda}-e^{-20\lambda}\approx0,173


1. e. De même qu'à la question 1.b,
P(X>18)=e^{-18\lambda}\approx0,067


2. a. Il s'agit de calculer P(20\le Y\le21).
1er cas : votre calculatrice vous permet de le faire, en rentrant les paramètres de la loi normale considérée.
2e cas : votre calculatrice ne connaît que la loi normale. Il faut donc s'y ramener. On sait que si Y suit une loi normale \mathcal{N}(\mu, \sigma^2), alors Z=\frac{Y-\mu}{\sigma} suit une loi normale centrée réduite. Ici,
P(20\le Y\le 21)=P\left(\dfrac{20-16}{1.95}\le\dfrac{Y-16}{1.95}\le\dfrac{21-16}{1.95}\right)=P\left(\dfrac{80}{39}\le Z\le\dfrac{100}{39}\right).

Dans tous les cas, on obtient P(20\le Y\le21)\approx0,015.

2. b. P((Y<11)\cup(Y>21))=1-P(11\le Y\le21). À l'aide de la calculatrice, on obtient P((Y<11)\cup(Y>21))\approx0,01.


Partie 2


Il est judicieux dans cette situation de dresser un arbre (partiel) de probabilités afin de faciliter la réflexion, et qui par ailleurs sera apprécié du correcteur.
Sujet de Mathématiques Bac S 2015 - Spé et non Spé : image 26


1. L'énoncé nous donne les probabilités que les bons d'achat rouges prennent les valeurs 30 ou 100 euros. La probabilité P_R(A) d'avoir un bon d'achat d'une valeur supérieure ou égale à 30 euros est donc P_R(A)=0,015+0,01=0,025.

2. Pour avoir un bon d'achat d'une valeur supérieure ou égale à 30 euros, il faut :
soit avoir un bon d'achat rouge d'une valeur supérieure ou égale à 30 euros,
soit avoir un bon d'achat vert d'une valeur supérieure ou égale à 30 euros.
La probabilité P(A) d'avoir un bon d'achat d'une valeur supérieure ou égale à 30 euros est, d'après la formule des probabilités totales :
P(A)=P(R)\times P_R(A)+P(V)\times P_V(A)=0,25\times0,025+0,75\times0,067=0,565\approx0,057.


3. Les doutes du directeur du magasin seraient justifiés si la fréquence obtenue était dans l'intervalle de fluctuation à 95%.
Ici, la probabilité de succès (avoir un bon de valeur supérieure à 30 euros) est p=0.057. La taille de l'échantillon est n=200. Or n\ge30,\ np>5,\ n(1-p)>5 donc on peut utiliser l'intervalle de fluctuation à 95% :
I=\left[p-1,96\times\cfrac{\sqrt{p(1-p)}}{\sqrt{n}},p+1,96\times\cfrac{\sqrt{p(1-p)}}{\sqrt{n}}\right].

Or p-1,96\times\cfrac{\sqrt{p(1-p)}}{\sqrt{n}}\le0.025 et p+1,96\times\cfrac{\sqrt{p(1-p)}}{\sqrt{n}} \ge0.08 donc
[0.025,0.08]\subset I.

La fréquence observée est \frac{6}{200}=0.03\in[0.025,0.08] donc \frac{6}{200}\in I donc les doutes du directeur ne sont pas justifiés.





exercice 2 - COMMUN À TOUS LES CANDIDATS


Pour que cette correction soit complète, on peut chercher à démontrer les résultats admis de l'énoncé. L'énoncé nous donne les vitesses auxquelles se déplacent les points M et N sur des droites. Démontrons l'expression des coordonnées, admises par l'énoncé.
M se déplace sur la droite (AB), dont \vec{i}\begin{pmatrix} 1\\0\\0\end{pmatrix} est un vecteur directeur donc pour tout t\ge0, il existe \alpha(t)\in\mathbb{R} tel que \overrightarrow{OM}=\overrightarrow{OA}+\alpha(t)\vec{i}. Alors, les coordonnées de M sont (x_A+ \alpha(t),y_A,z_A). Le vecteur vitesse du point M est le vecteur
\cfrac{\text{d}\overrightarrow{OM}}{\text{d}t}=\cfrac{\text{d}(\overrightarrow{OA}+ \overrightarrow{AM})}{\text{d}t}=\cfrac{\text{d}\overrightarrow{OA}}{\text{d}t}+\cfrac{\text{d} \overrightarrow{AM}}{\text{d}t}=\cfrac{\text{d} \overrightarrow{AM}}{\text{d}t}

qui a pour coordonnées \begin{pmatrix}\alpha'(t)\\0\\0\end{pmatrix}. La vitesse de M est la norme du vecteur vitesse, c'est donc \sqrt{\alpha'(t)^2}=|\alpha'(t)|. L'énoncé précise que le point se dirige de A vers B (\alpha'(t)\ge0) et que cette vitesse est constante égale à 1, donc pour tout t\ge0,\ \alpha'(t)=1. Cette équation différentielle impose que \alpha(t)=t+cc est une constante. Puisque M_0\equiv A, en t=0 on doit avoir \alpha(0)=0 donc c=0. Ainsi, on a montré que M_t a pour coordonnées (t,-1.5,5).
De même, N se déplace sur la droite (CD) dont \vec{u}\begin{pmatrix} 0\\4\\3\end{pmatrix} est un coefficient directeur. Ainsi, pour tout t\ge0, il existe \beta(t) tel que \overrightarrow{ON_t}=\overrightarrow{OC}+\beta(t)\vec{u}. Alors, les coordonnées de N_t sont (x_C,y_C+4\beta(t),z_C+3\beta(t)). Le vecteur vitesse du point N est, comme précédemment, le vecteur
\cfrac{\text{d}\overrightarrow{ON_t}}{\text{d}t}=\begin{pmatrix}0\\4\beta'(t)\\3\beta'(t)\end{pmatrix}.

La vitesse du point N_t est la norme du vecteur vitesse, soit \sqrt{(4\beta'(t))^2+(3\beta'(t))^2}=\sqrt{25\beta'(t)^2}=|5\beta'(t)|. Comme précédemment, on en déduit que \beta'(t)=\frac{1}{5}. Ainsi, \beta(t)=\frac{1}{5}\,t+cc est une constante. Comme N_0\equiv C, \beta(0)=0 et donc c=0. Ainsi, on a montré que N_t a pour coordonnées (11,0.8t,1+0.6t).


1. a. Un vecteur directeur de la droite (AB) est \begin{pmatrix}x_B-x_A\\y_B-y_A\\z_B-z_A\end{pmatrix}=\begin{pmatrix}1\\0\\0\end{pmatrix}. Les coefficients directeurs de (OI),(OJ),(OK) sont respectivement
\begin{pmatrix}1\\0\\0\end{pmatrix},\begin{pmatrix}0\\1\\0\end{pmatrix},\begin{pmatrix}0\\0\\1\end{pmatrix}

donc (AB) est parallèle à l'axe des abscisses (OI).

1. b. x_C=x_D donc (CD) est incluse dans le plan \mathcal{P} d'équation x=11. Ce plan est parallèle à (OJK) car \overrightarrow{OI} est normal à ces deux plans.

1. c. \overrightarrow{AE}\begin{pmatrix}x_E-x_A\\y_E-y_A\\z_E-z_A\end{pmatrix}=\begin{pmatrix} 11\\0\\0\end{pmatrix}=\cfrac{11}{2}\,\overrightarrow{AB} donc \overrightarrow{AE} et \overrightarrow{AB} sont colinéaires. Ainsi, E\in(AB). Par ailleurs, E vérifie x_E=11 donc E\in\mathcal{P}. Ainsi, (AB) coupe le plan \mathcal{P} au point E.
Par ailleurs, \overrightarrow{OI} est un vecteur normal à \mathcal{P} et \overrightarrow{OI} et \overrightarrow{AB} sont colinéaires, donc (AB) est orthogonale à \mathcal{P}.

Si la question avait été : "Déterminer l'intersection de (AB) et \mathcal{P}", on aurait pu procéder comme suit.
Comme \overrightarrow{OI} et \overrightarrow{AB} sont colinéaires, une représentation paramétrique de la droite (AB) est :
M(x,y,z)\in(AB)\Longleftrightarrow\exists t\in\mathbb{R},\ \left\lbrace\begin{array}{l}x=x_A+t=t\\y=y_A=-1\\z=z_A=5\end{array}\right.

Ainsi, M(x,y,z)\in(AB)\cap\mathcal{P}\Longleftrightarrow\exists t\in\mathbb{R},\ \left\lbrace\begin{array}{l}x=t\\y=-1\\z=5\\x=11\end{array}\right.\Longleftrightarrow M\equiv E(11,-1,5).


1. d. Un vecteur directeur de (CD) est \overrightarrow{CD}\begin{pmatrix}x_D-x_C\\y_D-y_C\\ z_D-z_C\end{pmatrix}=\begin{pmatrix}0\\4\\3\end{pmatrix} donc une représentation paramétrique de (CD) est
M(x,y,z)\in(CD)\Longleftrightarrow\exits t\in\mathbb{R},\ \left\lbrace\begin{array}{l}x=x_C=11\\y=y_C+4t=4t\\z=z_C+3t=1+3t\end{array}\right.

Ainsi, M(x,y,z)\in(AB)\cap(CD) si et seulement si les coordonnées vérifient les équations de ces droites : il existe t,t'\in\mathbb{R} tels que
\left\lbrace\begin{array}{ll}x=t&(1)\\y=-1&(2)\\z=5&(3)\\x=11&(4)\\y=4t'&(5)\\z=1+3t'&(6)\end{array}\right.

Les équations (2) et (5) imposent t'=\frac{-1}{4} alors que les équations (3) et (6) imposent t'=\frac{4}{3} donc il n'y a pas de solution.
Ainsi, (AB) et (CD) ne sont pas sécantes.

2. a. On écrit :
\begin{array}{rcl} M_tN_t^2&=&(x_{N_t}-x_{M_t})^2+(y_{M_t}-y_{N_t})^2+(z_{M_t}-z_{N_t})^2\\\\ &=&(11-t)^2+(\underbrace{0.8}_{\frac{4}{5}}t+1)^2+(\underbrace{0.6}_{\frac{3}{5}}t-4)^2\\\\ &=&121-22t+t^2+\frac{16}{25}\,t^2+\frac{8}{5}\,t+1+\frac{9}{25}\,t^2-\frac{24}{5}\,t+16\\\\ &=&2t^2-\frac{126}{5}\,t+138\\\\ &=&2t^2-25.2t+138 \end{array}


2. b. Le minimum d'un trinôme du second degré ax^2+bx+c (a>0) est atteint en x=\frac{-b}{2a}. Ici, la distance minimale est atteinte au temps t=\frac{25.2}{4}=12.6\text{ s} (et vaut 2t^2-25.2t+138=58.62\text{ cm}.)





exercice 3 - CANDIDATS N'AYANT PAS SUIVI L'ENSEIGNEMENT DE SPÉCIALITÉ


1. Le discriminant de cette équation est \Delta=8^2-4\times1\times64=-3\times64<0. L'équation a donc deux solutions complexes conjuguées z_1=\cfrac{8+i\sqrt{3\times64}}{2}=4+4i\sqrt{3} et z_2=4-4i\sqrt{3}.

2. a. |a|=\sqrt{16+16\times3}=8 et a=8\left(\cfrac{1}{2}+i\,\cfrac{\sqrt{3}}{2}}\right)=8e^{i\frac{\pi}{3}} donc un argument de a est \cfrac{\pi}{3}.

2. b. On a vu que a=8e^{i\frac{\pi}{3}} donc b=\overline{a}=8e^{-i\frac{\pi}{3}}.

2. c. |c|=8=|a|=|b| donc OA=OB=OC et les points A, B et C sont sur le cercle de centre O et de rayon 8.

2.d. On trace le cercle de centre O et de rayon 8 puis on utilise les abscisses des points d'affixe a,b,c (qui sont connues précisément), pour placer A,B et C.
Sujet de Mathématiques Bac S 2015 - Spé et non Spé : image 25


3. a. b'=be^{i\frac{\pi}{3}}=8e^{i(-\frac{\pi}{3}+\frac{\pi}{3})}=8.

3. b. |a'|=|ae^{i\frac{\pi}{3}}|=|a|\times|e^{i\frac{\pi}{3}}|=|a|=8 et a'=8e^{i(\frac{\pi}{3}+\frac{\pi}{3})}= 8e^{\frac{2i\pi}{3}} donc un argument de a' est \cfrac{2\pi}{3}.

Un calcul simple montre que a'=-4+4i\sqrt{3} et c'=-4\sqrt{3}+4i.

4. Si M(x_M,y_M) et N(x_N,y_N) sont deux points du plan, alors m=x_M+iy_M et n=x_N+iy_N. On sait que le milieu du segment [MN] est le point de coordonnées (\frac{x_M+y_M}{2},\frac{y_M+y_N}{2}), soit le point d'affixe \frac{x_M+x_N}{2}+i\,\frac{y_M+y_N}{2}=\frac{m+n}{2}.
De plus, MN=\sqrt{(x_N-x_M)^2+(y_N-y_M)^2}=|n-m|.

4. a. On calcule :
r=\frac{a'+b}{2}=\frac{-4+4i\sqrt{3}+4-4i\sqrt{3}}{2}=0

s=\frac{b'+c}{2}=\frac{8+8i}{2}=4+4i

t=\frac{c'+a}{2}=\frac{-4\sqrt{3}+4i+4+4i\sqrt{3}}{2}=2-2\sqrt{3}+i(2+2\sqrt{3})


4. b. Pour pouvoir faire une conjecture, on a besoin de faire un dessin :
Sujet de Mathématiques Bac S 2015 - Spé et non Spé : image 28

On peut donc conjecturer que le triangle RST est équilatéral.
Pour le démontrer, il suffit de montrer que SR=ST=TR soit |r-s|=|t-s|=|r-t|. Or
|r-s|=|s|=\sqrt{4^2+4^2}=\sqrt{16\times2}=4\sqrt{2}

|t-s|=\sqrt{(2-2\sqrt{3}-4)^2+(2+2\sqrt{3}-4)^2}=\sqrt{4+8\sqrt{3}+12+4-8\sqrt{3}+12}=\sqrt{16\times2}=4\sqrt{2}

|r-t|=|t|=\sqrt{(2-2\sqrt{3})^2+(2+2\sqrt{3})^2}=\sqrt{4-8\sqrt{3}+12+4+8\sqrt{3}+12}=4\sqrt{2}

donc la conjecture est démontrée.





exercice 3 - CANDIDATS AYANT SUIVI L'ENSEIGNEMENT DE SPÉCIALITÉ


1. a. 7\times3-5\times4=21-20=1 donc (3,4) est solution.

1. b. Soit (x,y) une solution de (E). Alors
7x-5y=1
et on a vu que
7\times3-5\times4=1.

En soustrayant ces deux égalités, on obtient :
7x-5y-7\times3+5\times4=0

soit
7(x-3)=5(y-4).

Réciproquement, si 7(x-3)=5(y-4) alors en simplifiant les constantes, il vient 7x-5y=1.
Ainsi, (x,y)\in\mathbb{Z}^2 est solution si et seulement si 7(x-3)=5(y-4).

1. c. Soit (x,y) un couple d'entiers solution. Alors, d'après la question précédente, 7(x-3)=5(y-4). On en déduit, comme 7 et 5 sont premiers entre eux, que 7 divise y-4. Ainsi, il existe k\in\mathbb{Z} tel que y-4=7k, soit y=7k+4. En reportant ceci dans l'équation 7x-5y=1, on obtient  7x-35k-20=1, soit x=5k+3.
Ainsi, l'ensemble des solutions entières de (E) est inclus dans l'ensemble \lbrace(x,y)\in\mathbb{Z}^2,\exists k\in\mathbb{Z},x=5k+3,y=k+4\rbrace.
Réciproquement, pour k\in\mathbb{Z}, 7\times(5k+3)-5(7k+4)=1 donc (5k+3,7k+4) est solution de (E).
On a donc montré que les solutions entières de (E) sont exactement les couples (x,y) d'entiers relatifs tel que
\left\lbrace\begin{array}{l}x=5k+3\\y=7k+4\end{array}\right., k\in\mathbb{Z}.


2. Les nombres x et y de jetons rouges ou verts vérifient l'équation (E) donc k\in\mathbb{Z} tel que x=5k+3 et y=7k+3 avec k un entier relatif tel que x,y\ge0 et x+y\le25.
On obtient ainsi les solutions
k=0:\qquad(x,y)=(3,4)
et
k=1:\qquad(x,y)=(8,11).

Ainsi, il y a 3 jetons rouges, 4 jetons verts et 25-4-3=18 jetons blancs ; ou 8 jetons rouges, 11 jetons verts et 25-8-11=6 jetons blancs.
Les autres valeurs de k fournissent des solutions de (E) qui ne satisfont pas les autres conditions.

3. Au départ, le pion est sur le sommet A donc X_0=\begin{pmatrix}1&0&0\end{pmatrix}.
Soit n\in\mathbb{N}. Pour être en A à l'étape n+1, à l'étape n on peut avoir été en A et avoir tiré un jeton blanc, avoir été en B et avoir tiré un jeton rouge ou avoir été en C et avoir tiré un jeton rouge. Ainsi,
a_{n+1}=\frac{18}{25}a_n+\frac{3}{25}b_n+\frac{3}{25}c_n=0.72a_n+0.12b_n+0.12c_n.

De même, on exprime b_{n+1} et c_{n+1} en fonction de a_n,b_n,c_n. On obtient :
\left\lbrace\begin{array}{rcl}a_{n+1}&=&0.72a_n+0.12b_n+0.12c_n\\b_{n+1}&=&0.12a_n+0.72b_n+0.16c_n\\ c_{n+1}&=&0.16a_n+0.16b_n+0.72c_n\end{array}\right.

On peut écrire ceci matriciellement (avec le choix de l'énoncé d'utiliser une matrice ligne...) :
X_{n+1}=\begin{pmatrix}a_n&b_n&c_n\end{pmatrix}\begin{pmatrix}0.72&0.12&0.16\\0.12&0.72&0.16\\0.12& 0.16&0.72\end{pmatrix}=X_nT.

4. a. On utilise la calculatrice pour trouver l'inverse de P^{-1} : P=\begin{pmatrix}1&7&4\\1&-3&4\\1&-3&-7\end{pmatrix}.

4. b. Montrons par récurrence sur n\in\mathbb{N} que T^n=PD^nP^{-1}.
n=0 : D^0=I_3 donc I_3=T^0=PI_3P^{-1}=PP^{-1}=I_3
n=1 : Par hypothèse de l'énoncé T=PDP^{-1}.
Soit n\ge1 tel que T^n=PD^nP^{-1}.
Alors, par hypothèse de récurrence, T^{n+1}=T\times T^n=TPD^nP^{-1}. Or T=PDP^{-1} donc on obtient T^{n+1}=PDP^{-1}PD^nP^{-1}=PD^{n+1}P^{-1}.
Ainsi, d'après le principe de récurrence, \forall n\in\mathbb{N},\ T^n=PD^nP^{-1}.

4. c. On montre par récurrence sur n\in\mathbb{N} que D^n=\begin{pmatrix}1&0&0\\ 0&0.6^n&0\\0&0&0.56^n\end{pmatrix}.

On calcule les coefficients \alpha_n et \beta_n en effectuant le produit T^n=PD^nP^{-1}.

5. a. On montre par récurrence sur n\in\mathbb{N} que X_n=X_0T^n. Soit n\in \mathbb{N}. \begin{pmatrix}a_n&b_n&c_n\end{pmatrix}=\begin{pmatrix}1&0&0\end{pmatrix}\begin{pmatrix} \alpha_n&\beta_n&\gamma_n\\\dots&\dots&\dots\\\dots&\dots&\dots\end{pmatrix}=\begin{pmatrix}\alpha_n&\beta_n&\gamma_n\end{pmatrix}.
Ainsi, a_n=\alpha_n,\ b_n=\beta_n et c_n=1-a_n-b_n=1-\alpha_n-\beta_n.

5. b. Comme -1<0.6<1,\ \displaystyle\lim_{n\to+\infty}0.6^n=0 donc (a_n) a une limite et \displaystyle\lim_{n\to+\infty}a_n=\lim_{n\to+\infty}\cfrac{3}{10}+\cfrac{7}{10}\times0.6^n=\cfrac{3}{10}.
De même, comme -1<0.56<0.6<1, (b_n) a une limite et \displaystyle\lim_{n\to+\infty}b_n=\cfrac{37}{110}.
Enfin, (c_n) a une limite et \displaystyle\lim_{n\to+\infty}c_n=1-\cfrac{3}{10}-\cfrac{37}{110}=\cfrac{4}{11}.

5. c. La limite de (c_n) est supérieure aux limites de (a_n) et (b_n) donc, après un grand nombre d'itérations, la probabilité c_n sera supérieure à a_n et b_n et donc on aura plus de chance de se trouver sur le sommet C.





exercice 4 - COMMUN À TOUS LES CANDIDATS


Partie 1


1. Les fonctions x\mapsto x+1,x\mapsto-3x+7,x\mapsto\ln(x+1) sont dérivables sur l'intervalle [0,20] donc f est dérivable et
\forall x\in[0,20],\qquad f'(x)=1\times\ln(x+1)+(x+1)\times\cfrac{1}{x+1}-3=\ln(x+1)-2.


2. Par croissance (et bijectivité pour l'équivalence) de la fonction exponentielle, \ln(x+1)-2\ge0 \Longleftrightarrow x+1\ge e^2\Longleftrightarrow x\ge e^2-1. Ainsi, f est décroissante sur [0, e^2-1] et croissante sur [e^2-1,20]. On peut dresser le tableau de variations :
Sujet de Mathématiques Bac S 2015 - Spé et non Spé : image 27


3. Le coefficient directeur de la tangente à la courbe au point d'abscisse 0 est f'(0)=-2.

4. Si vous vous demandez d'où vient la primitive de x\mapsto(x+1)\ln(x+1) donnée par l'énoncé (la fonction g), renseignez-vous sur la méthode d'intégration par parties, qui a été retirée des derniers programmes de Terminale (ici, ce n'est pas un exemple facile, n'hésitez pas à poser des questions sur le forum).
Pour trouver une primitive de f, il faut une primitive de x\mapsto(x+1)\ln(x+1). Comme, avec les outils du programme, on ne sait pas en calculer, elle est donnée par l'énoncé, c'est g. Il faut également une primitive de x\mapsto-3x+7. On vérifie que x\mapsto-3\times\cfrac{x^2}{2}+7x en est une.
Ainsi, la fonction F:x\mapsto\cfrac{1}{2}(x+1)^2\ln(x+1)-\cfrac{1}{4}x^2-\cfrac{1}{2}x-\cfrac{3}{2}x^2+7x=\cfrac{1}{2} (x+1)^2\ln(x+1)-\cfrac{7}{4}\,x^2+\cfrac{13}{2}\,x est une primitive de f sur l'intervalle [0,20] (celle-ci s'annule en 0).


Partie 2


1. P1 : Comme f est décroissante sur [0,e^2-1], le maximum de f sur cet intervalle est f(0)=7. Comme f est croissante sur [e^2-1,20], le maximum de f sur cet intervalle est f(20). Ainsi, le maximum de f sur [0,20] est \max( 7,f(20))=\max(7,21\ln(21)-53)=21\ln(21)-53.
De plus, le minimum de f est f(e^2-1)=10-e^2.
La différence de hauteur entre le point le plus haut et le point le plus bas de la piste est donc 21\ln(21)-53-(10-e^2) =21\ln(21)+e^2-63\ge8 donc l'affirmation est exacte.

P2 : L'inclinaison en B est |f'(0)|=2 et l'inclinaison en C est |f'(20)|=\ln(21)-2. Or 2|f'(20)|\approx2,09. Ainsi, l'affirmation est exacte.
Précisons toutefois que cette question n'a, mathématiquement, aucun sens puisque "presque" ne veut rien dire. Il faudrait préciser un niveau de tolérance. Une façon de faire est de calculer \frac{2|f'(20)|-|f'(0)|}{|f'(0)|}\approx0.045, et conclure que l'erreur est inférieure à 5%, ce qui peut être le niveau de tolérance fixé.

2. L'aire latérale gauche est \mathcal{A}_1=OB\times OA=f(0)\times DD'=70\text{ m}^2.
L'aire latérale droite est \mathcal{A}_2=DC\times DD'=f(20)\times DD'=210\ln(21)-530.
Les aires latérales des faces avant et arrière sont égales et valent \mathcal{A}_3=\displaystyle\int_0^{20}f(t)\text{d}t=F(20) -F(0)=\frac{441\ln(21)}{2}-570.
Ainsi, l'aire à peindre est \mathcal{A}=\mathcal{A}_1+\mathcal{A}_2+2\mathcal{A}_3 et le volume de peinture nécessaire est V=\cfrac{\mathcal{A}}{5}=\cfrac{651\ln(21)}{5}-320\approx76.4\text{ L} donc il faut prévoir 77 litres de peinture.

3. a. Pour tout 0\le k\le19,\ B_kB_{k+1}=\sqrt{(x_{B_{k+1}}-x_{B_k})^2+(y_{B_{k+1}}-y_{B_k})^2}= \sqrt{1+(f(k+1)-f(k))^2}.

3. b. Il faut sommer les aires des rectangles B_kB_{k+1}B_{k+1}'B_k' pour k allant de 0 à 19. Pour chaque k, l'aire du rectangle est B_kB_{k+1}\times B_kB_k'=B_kB_{k+1}\times10. On peut donc compléter l'algorithme de l'énoncé :
\begin{array}{|l|l|} \hline \text{Variables}&S\text{ : réel}\\ &K\text{ : entier}\\ \text{Fonction}&f\text{ : définie par }f(x)=(x+1)\ln(x+1)-3x+7\\ \hline \text{Traitement}\qquad\qquad&S\text{ prend pour valeur 0}\\ &\text{Pour }K\text{ variant de 0 à 19}\\ &\quad S\text{ prend pour valeur }S+10\sqrt{1+(f(k+1)-f(k))^2}\\ &\text{FinPour}\\ \hline \text{Sortie}&\text{Afficher }S\\ \hline \end{array}



Publié le
ceci n'est qu'un extrait
Pour visualiser la totalité des cours vous devez vous inscrire / connecter (GRATUIT)
Inscription Gratuite se connecter
Merci à
david9333
pour avoir contribué à l'élaboration de cette fiche


Vous devez être membre accéder à ce service...

Pas encore inscrit ?

1 compte par personne, multi-compte interdit !

Ou identifiez-vous :


Rester sur la page

Inscription gratuite

Fiches en rapport

parmi 1674 fiches de maths

Désolé, votre version d'Internet Explorer est plus que périmée ! Merci de le mettre à jour ou de télécharger Firefox ou Google Chrome pour utiliser le site. Votre ordinateur vous remerciera !